2
$\begingroup$

This is a continuation of the previous question Comparison between first minimum of a lattice and a discrete subgroup in function field. Let $\mathbb{F}_q(T)$ denote the function field over $\mathbb{F}_q$, where $q$ is a prime power. The norm in this field is defined by

$ \left| \frac{f}{g} \right| = q^{\deg(f) - \deg(g)}, $ for $f, g \in \mathbb{F}_q[T^{-1}]$. Consider the field of Laurent series over $\mathbb{F}_q$, denoted by $\mathbb{F}_q((T^{-1}))$. It is well known that the set $ g \mathbb{F}_q[T]^d $ forms a lattice in the vector space $\mathbb{F}_q((T^{-1}))^d$, where $g \in SL(d, \mathbb{F}_q((T^{-1})))$.

Now, let us extend the field $\mathbb{F}_q((T^{-1}))$ by adjoining $T^{\frac{1}{2}}$, and denote this extension by $K$. In this setting, the ring extension $\mathbb{F}_q[T^{\frac{1}{2}}]$ forms a lattice in $K$, whereas $\mathbb{F}_q[T]$ does not. Given $\frac{f_1}{g_1}, \frac{f_2}{g_2} \in \mathbb{F}_q(T)^2$, we have $(f_1, f_2, g) \in \mathbb{F}_q[T]^3$.

Consider the flow $ g_t = \operatorname{diag}\left(T^{\frac{1}{2}}, T^{\frac{1}{2}}, T^{-1}\right) $ acting on $K^3$, and denote by $g_t \mathbb{F}_q[T^{\frac{1}{2}}]^3$ and $g_t \mathbb{F}_q[T]^3$ the corresponding lattice and discrete subgroup, respectively, in $K$. As observed in the previous discussion, the first successive minima coincide: $ \lambda_1\bigl(g_t \mathbb{F}_q[T^{\frac{1}{2}}]^3\bigr) = \lambda_1\bigl(g_t \mathbb{F}_q[T]^3\bigr). $ The question I pose is whether anything can be said about the $i$-th successive minima.

Since $\mathbb{F}_q[T] \subset \mathbb{F}_q[T^{\frac{1}{2}}]$, it follows that

$ \lambda_i\bigl(g_t \mathbb{F}_q[T^{\frac{1}{2}}]^3\bigr) \leq \lambda_i\bigl(g_t \mathbb{F}_q[T]^3\bigr). $ However, is it possible to make this inequality more explicit by identifying a constant, potentially depending on the flow or the degree of the field extension? Given that Minkowski's theorem does not hold for discrete subgroups in this context, I am uncertain whether such a comparison can be established.

$\endgroup$

0

You must log in to answer this question.

Start asking to get answers

Find the answer to your question by asking.

Ask question

Explore related questions

See similar questions with these tags.